Variance of Stochastic Integral $int_0^1t^2 dWt$Expected value of the stochastic integral $int_0^t e^as dW_s$Expectation and variance of this stochastic processVariance of Ito IntegralFind a process $f=f(t,W_t)$ such that another process is a martingaleStochastic Calculus - Ito decompositionIto formula - some doubtsDistribution of stochastic integralWhat are the components in the Ito Process?Representation of Ito integralCalculating Variance of the stochastic process

How did Arya survive the stabbing?

How to safely derail a train during transit?

Why escape if the_content isnt?

How do I find the solutions of the following equation?

How to check is there any negative term in a large list?

How does Loki do this?

Are student evaluations of teaching assistants read by others in the faculty?

How to be diplomatic in refusing to write code that breaches the privacy of our users

How did Doctor Strange see the winning outcome in Avengers: Infinity War?

Why not increase contact surface when reentering the atmosphere?

What is the difference between "behavior" and "behaviour"?

Tiptoe or tiphoof? Adjusting words to better fit fantasy races

Anatomically Correct Strange Women In Ponds Distributing Swords

Nautlius: add mouse right-click action to compute MD5 sum

CREATE opcode: what does it really do?

Do sorcerers' subtle spells require a skill check to be unseen?

Applicability of Single Responsibility Principle

Lay out the Carpet

Why Were Madagascar and New Zealand Discovered So Late?

Was Spock the First Vulcan in Starfleet?

How to escape string to filename? It is in backup a file append date

Gears on left are inverse to gears on right?

Purchasing a ticket for someone else in another country?

System.debug(JSON.Serialize(o)) Not longer shows full string



Variance of Stochastic Integral $int_0^1t^2 dWt$


Expected value of the stochastic integral $int_0^t e^as dW_s$Expectation and variance of this stochastic processVariance of Ito IntegralFind a process $f=f(t,W_t)$ such that another process is a martingaleStochastic Calculus - Ito decompositionIto formula - some doubtsDistribution of stochastic integralWhat are the components in the Ito Process?Representation of Ito integralCalculating Variance of the stochastic process













0












$begingroup$


I want to find Variance of integral $int_0^1t^2 dWt$



$W_t$ is Brownian motion



What I did:



I used Ito formula and got:



$int_0^1t^2 dWt = W_1 - int_0^1 2tW_t dt$ (correct me if I'm wrong)



I do not know how to compute $int_0^1 2tW_t dt$ or compute variance for whole answer.










share|cite|improve this question











$endgroup$







  • 2




    $begingroup$
    by Ito the variance is $int_0^1 t^4 dt$
    $endgroup$
    – Math-fun
    yesterday















0












$begingroup$


I want to find Variance of integral $int_0^1t^2 dWt$



$W_t$ is Brownian motion



What I did:



I used Ito formula and got:



$int_0^1t^2 dWt = W_1 - int_0^1 2tW_t dt$ (correct me if I'm wrong)



I do not know how to compute $int_0^1 2tW_t dt$ or compute variance for whole answer.










share|cite|improve this question











$endgroup$







  • 2




    $begingroup$
    by Ito the variance is $int_0^1 t^4 dt$
    $endgroup$
    – Math-fun
    yesterday













0












0








0





$begingroup$


I want to find Variance of integral $int_0^1t^2 dWt$



$W_t$ is Brownian motion



What I did:



I used Ito formula and got:



$int_0^1t^2 dWt = W_1 - int_0^1 2tW_t dt$ (correct me if I'm wrong)



I do not know how to compute $int_0^1 2tW_t dt$ or compute variance for whole answer.










share|cite|improve this question











$endgroup$




I want to find Variance of integral $int_0^1t^2 dWt$



$W_t$ is Brownian motion



What I did:



I used Ito formula and got:



$int_0^1t^2 dWt = W_1 - int_0^1 2tW_t dt$ (correct me if I'm wrong)



I do not know how to compute $int_0^1 2tW_t dt$ or compute variance for whole answer.







stochastic-processes stochastic-calculus stochastic-integrals






share|cite|improve this question















share|cite|improve this question













share|cite|improve this question




share|cite|improve this question








edited 10 hours ago







mahdi

















asked yesterday









mahdimahdi

7412




7412







  • 2




    $begingroup$
    by Ito the variance is $int_0^1 t^4 dt$
    $endgroup$
    – Math-fun
    yesterday












  • 2




    $begingroup$
    by Ito the variance is $int_0^1 t^4 dt$
    $endgroup$
    – Math-fun
    yesterday







2




2




$begingroup$
by Ito the variance is $int_0^1 t^4 dt$
$endgroup$
– Math-fun
yesterday




$begingroup$
by Ito the variance is $int_0^1 t^4 dt$
$endgroup$
– Math-fun
yesterday










1 Answer
1






active

oldest

votes


















3












$begingroup$

Expectation of Ito integral of a deterministic function wrt to a Brownian motion is $0$. Thus,



$$ mathbbVarleft[int_0^1 t^2 d W_t right]= mathbbEleft[left(int_0^1 t^2 d W_tright)^2right]$$



Applying Ito Isometry, we obtain



$$ mathbbEleft[left(int_0^1 t^2 d W_tright)^2right] = mathbbEleft[int_0^1 t^4 dtright] = int_0^1 t^4 dt = frac15 $$






share|cite|improve this answer









$endgroup$












    Your Answer





    StackExchange.ifUsing("editor", function ()
    return StackExchange.using("mathjaxEditing", function ()
    StackExchange.MarkdownEditor.creationCallbacks.add(function (editor, postfix)
    StackExchange.mathjaxEditing.prepareWmdForMathJax(editor, postfix, [["$", "$"], ["\\(","\\)"]]);
    );
    );
    , "mathjax-editing");

    StackExchange.ready(function()
    var channelOptions =
    tags: "".split(" "),
    id: "69"
    ;
    initTagRenderer("".split(" "), "".split(" "), channelOptions);

    StackExchange.using("externalEditor", function()
    // Have to fire editor after snippets, if snippets enabled
    if (StackExchange.settings.snippets.snippetsEnabled)
    StackExchange.using("snippets", function()
    createEditor();
    );

    else
    createEditor();

    );

    function createEditor()
    StackExchange.prepareEditor(
    heartbeatType: 'answer',
    autoActivateHeartbeat: false,
    convertImagesToLinks: true,
    noModals: true,
    showLowRepImageUploadWarning: true,
    reputationToPostImages: 10,
    bindNavPrevention: true,
    postfix: "",
    imageUploader:
    brandingHtml: "Powered by u003ca class="icon-imgur-white" href="https://imgur.com/"u003eu003c/au003e",
    contentPolicyHtml: "User contributions licensed under u003ca href="https://creativecommons.org/licenses/by-sa/3.0/"u003ecc by-sa 3.0 with attribution requiredu003c/au003e u003ca href="https://stackoverflow.com/legal/content-policy"u003e(content policy)u003c/au003e",
    allowUrls: true
    ,
    noCode: true, onDemand: true,
    discardSelector: ".discard-answer"
    ,immediatelyShowMarkdownHelp:true
    );



    );













    draft saved

    draft discarded


















    StackExchange.ready(
    function ()
    StackExchange.openid.initPostLogin('.new-post-login', 'https%3a%2f%2fmath.stackexchange.com%2fquestions%2f3163682%2fvariance-of-stochastic-integral-int-01t2-dwt%23new-answer', 'question_page');

    );

    Post as a guest















    Required, but never shown

























    1 Answer
    1






    active

    oldest

    votes








    1 Answer
    1






    active

    oldest

    votes









    active

    oldest

    votes






    active

    oldest

    votes









    3












    $begingroup$

    Expectation of Ito integral of a deterministic function wrt to a Brownian motion is $0$. Thus,



    $$ mathbbVarleft[int_0^1 t^2 d W_t right]= mathbbEleft[left(int_0^1 t^2 d W_tright)^2right]$$



    Applying Ito Isometry, we obtain



    $$ mathbbEleft[left(int_0^1 t^2 d W_tright)^2right] = mathbbEleft[int_0^1 t^4 dtright] = int_0^1 t^4 dt = frac15 $$






    share|cite|improve this answer









    $endgroup$

















      3












      $begingroup$

      Expectation of Ito integral of a deterministic function wrt to a Brownian motion is $0$. Thus,



      $$ mathbbVarleft[int_0^1 t^2 d W_t right]= mathbbEleft[left(int_0^1 t^2 d W_tright)^2right]$$



      Applying Ito Isometry, we obtain



      $$ mathbbEleft[left(int_0^1 t^2 d W_tright)^2right] = mathbbEleft[int_0^1 t^4 dtright] = int_0^1 t^4 dt = frac15 $$






      share|cite|improve this answer









      $endgroup$















        3












        3








        3





        $begingroup$

        Expectation of Ito integral of a deterministic function wrt to a Brownian motion is $0$. Thus,



        $$ mathbbVarleft[int_0^1 t^2 d W_t right]= mathbbEleft[left(int_0^1 t^2 d W_tright)^2right]$$



        Applying Ito Isometry, we obtain



        $$ mathbbEleft[left(int_0^1 t^2 d W_tright)^2right] = mathbbEleft[int_0^1 t^4 dtright] = int_0^1 t^4 dt = frac15 $$






        share|cite|improve this answer









        $endgroup$



        Expectation of Ito integral of a deterministic function wrt to a Brownian motion is $0$. Thus,



        $$ mathbbVarleft[int_0^1 t^2 d W_t right]= mathbbEleft[left(int_0^1 t^2 d W_tright)^2right]$$



        Applying Ito Isometry, we obtain



        $$ mathbbEleft[left(int_0^1 t^2 d W_tright)^2right] = mathbbEleft[int_0^1 t^4 dtright] = int_0^1 t^4 dt = frac15 $$







        share|cite|improve this answer












        share|cite|improve this answer



        share|cite|improve this answer










        answered 15 hours ago









        MdocMdoc

        556515




        556515



























            draft saved

            draft discarded
















































            Thanks for contributing an answer to Mathematics Stack Exchange!


            • Please be sure to answer the question. Provide details and share your research!

            But avoid


            • Asking for help, clarification, or responding to other answers.

            • Making statements based on opinion; back them up with references or personal experience.

            Use MathJax to format equations. MathJax reference.


            To learn more, see our tips on writing great answers.




            draft saved


            draft discarded














            StackExchange.ready(
            function ()
            StackExchange.openid.initPostLogin('.new-post-login', 'https%3a%2f%2fmath.stackexchange.com%2fquestions%2f3163682%2fvariance-of-stochastic-integral-int-01t2-dwt%23new-answer', 'question_page');

            );

            Post as a guest















            Required, but never shown





















































            Required, but never shown














            Required, but never shown












            Required, but never shown







            Required, but never shown

































            Required, but never shown














            Required, but never shown












            Required, but never shown







            Required, but never shown







            Popular posts from this blog

            Triangular numbers and gcdProving sum of a set is $0 pmod n$ if $n$ is odd, or $fracn2 pmod n$ if $n$ is even?Is greatest common divisor of two numbers really their smallest linear combination?GCD, LCM RelationshipProve a set of nonnegative integers with greatest common divisor 1 and closed under addition has all but finite many nonnegative integers.all pairs of a and b in an equation containing gcdTriangular Numbers Modulo $k$ - Hit All Values?Understanding the Existence and Uniqueness of the GCDGCD and LCM with logical symbolsThe greatest common divisor of two positive integers less than 100 is equal to 3. Their least common multiple is twelve times one of the integers.Suppose that for all integers $x$, $x|a$ and $x|b$ if and only if $x|c$. Then $c = gcd(a,b)$Which is the gcd of 2 numbers which are multiplied and the result is 600000?

            Ingelân Ynhâld Etymology | Geografy | Skiednis | Polityk en bestjoer | Ekonomy | Demografy | Kultuer | Klimaat | Sjoch ek | Keppelings om utens | Boarnen, noaten en referinsjes Navigaasjemenuwww.gov.ukOffisjele webside fan it regear fan it Feriene KeninkrykOffisjele webside fan it Britske FerkearsburoNederlânsktalige ynformaasje fan it Britske FerkearsburoOffisjele webside fan English Heritage, de organisaasje dy't him ynset foar it behâld fan it Ingelske kultuergoedYnwennertallen fan alle Britske stêden út 'e folkstelling fan 2011Notes en References, op dizze sideEngland

            Հադիս Բովանդակություն Անվանում և նշանակություն | Դասակարգում | Աղբյուրներ | Նավարկման ցանկ